PrepTest 87 - June 19

You might also like

Download as pdf or txt
Download as pdf or txt
You are on page 1of 34

-1-

SECTION I
Time—35 minutes
27 Questions

Directions: Each set of questions in this section is based on a single passage or a pair of passages. The questions are to be answered
on the basis of what is stated or implied in the passage or pair of passages. For some questions, more than one of the choices could
conceivably answer the question. However, you are to choose the best answer; that is, choose the response that most accurately and
completely answers the question and mark that response on your answer sheet.

Most writings on the subject of motion based on original, uncompromised versions of the films,
pictures, including those scrutinizing the structural they may raise false expectations with regard to the
characteristics, aesthetic qualities, and effects of more or less faulty versions that are often available to
motion pictures on audiences, have traditionally been viewers.
(5) relatively abstract and have not considered what a
film’s audiences actually see. In fact, various external 1. In the passage, the author primarily attempts to
factors intervene between the filmmaker’s intent and the
audience’s experience, often altering the qualities of a (A) provide evidence against a claim that is often
film and, consequently, the viewer’s perception of it. made in the criticism of a particular art form
(10) In the process of distribution, a film can be (B) establish that changing the materials used in
mutilated in many ways. The damage is most obvious a particular art form would enhance public
when films in one language are shown to audiences that appreciation of that art form
speak a different language. Subtitling may be simply (C) refute a commonly held view regarding the
incompetent, full of mistakes, or used for actual detrimental effects of criticism on a particular
(15) censorship. Dubbing—a significantly more profound art form
intervention—can be even more damaging. Some films (D) describe a problem that is generally overlooked
are reedited to render them “more understandable” by in the criticism of a particular art form
their target audiences, while others are given new titles (E) explain why a particular art form is the target of
rather than translations of their original titles, a practice negative criticism
(20) that often creates false expectations and distorts the
work’s intent. 2. The author distinguishes film from at least some other art
When a film is shown on television or video, it forms with regard to the
suffers the most extensive deformations. In addition to (A) extent of public reliance on professional analyses
causing a loss of image size and definition, current and evaluations
(25) mass-market television and video technology is harmful (B) possibility of creating multiple instances of the
in other ways. These intrusions include advertisements same artwork
that break the intended continuity, the superimposition (C) susceptibility of the artwork to damage through
of images—such as station identifications and weather environmental factors and aging
bulletins—over parts of the picture, and spoken (D) degree of control that a work’s creator has over
(30) announcements over parts of the soundtrack considered the conditions of its public presentation
by programmers to be “unimportant.” Some alterations, (E) complexity of the methods used to provide public
such as a subtle increase in the projection speed of a access to the artwork
televised movie to obtain more commercial time, are
almost imperceptible but nonetheless detrimental to the
(35) integrity of the film.
It seems that audiences and even most film critics
have tacitly accepted this situation—they rarely speak GO ON TO THE NEXT PAGE.
about it. This may be partly because of the special
nature of film. In many other arts it is obvious that
(40) reproductions of a work are not the work itself, and they
are not treated as such. However, the very nature of film
makes it an exactly reproducible art form, under ideal
conditions, each print is not merely a reproduction but
is in fact another instance of the work itself. But we
(45) tend to overlook how rarely the ideal conditions apply,
and this is disturbing for two reasons. First, professional
analysis, interpretation, and evaluation may be unfair to
filmmakers when—as is surprisingly often the case—
they are based on a version that has already been
(50) seriously altered. Second, when critics’ comments are

PrepTest 87
-2-

3. It can be inferred from the passage that the author would 5. It can be inferred from the passage that the author
be most likely to agree with which one of the following believes which one of the following statements about
statements? films?
(A) Films should be projected at precisely the speed (A) When shown on television—even without having
at which they were designed to be projected. been reedited and without any commercial
(B) Filmmakers should accept the fact that criticisms breaks or superimposed messages—films can be
are often directed against distributors rather than artistically compromised to some extent.
against filmmakers. (B) Films are considered by many art critics to
(C) Film critics should acknowledge that mutilations be of questionable significance as a topic of
of films during distribution and public exhibition serious scholarly analysis, interpretation, and
are inevitable. evaluation.
(D) Film commentaries should not be concerned with (C) Because of technical limitations involved in the
audience reactions to films. process of reproducing films, no reproduction
(E) Films should be viewed only in relatively large, of any one film can be strictly classified as the
darkened theaters. work that the filmmaker intended.
(D) Even when they are distributed in
4. Which one of the following would, if true, most uncompromised versions, films elicit variable
strengthen the author’s argument concerning any responses from viewers, and thus they can rarely
modification of a film for distribution? be expected to have the aesthetic impact that the
filmmakers intend.
(A) Almost all filmmakers whose work critics respect (E) Most films do not meet the standards set by
approve of the distribution of altered versions of writings that analyze their structural and
their films. aesthetic qualities.
(B) Mass-market television and video technology
has recently improved in its ability to present 6. Suppose that a Russian company is planning to distribute
films in ways that conform to the intentions of to Russian-speaking audiences a film that was produced
filmmakers. in Italy with dialogue originally spoken only in Italian.
(C) Many professional commentaries on nonfilm It can be inferred from the passage that the author would
artworks are based on aged, mutilated, or be most likely to agree with which one of the following
otherwise altered versions of those works. recommendations regarding the film?
(D) Almost all viewers of films are unaware of the
professional commentaries that are written about (A) The company should attempt to translate the
those films. film’s title into Russian but should also make
(E) In almost every film that has a soundtrack, all explanatory material available for audiences that
parts of the soundtrack are designed by the know no Italian.
filmmaker to contribute significantly to the (B) The company should first make the film available
film’s artistic value. only to critics who understand Italian and
afterwards should release it to the general
public.
(C) The film should be subtitled in Russian, but
only if the translation is strictly faithful to the
original.
(D) The version of the film that Russian-speaking
audiences see should not include subtitles.
(E) The film should be very cautiously restructured,
as needed, to make the filmmaker’s intentions
clearer to Russian-speaking audiences.

GO ON TO THE NEXT PAGE.

PrepTest 87
-3-

A recent worldwide decline of ocean fishery stocks 7. Which one of the following most accurately describes the
has stimulated rapid growth in cultivated production main point of the passage?
of fish and shellfish, usually known as fish farming.
Between 1987 and 1997, for example, global (A) The recent decline of ocean fishery stocks was
(5) fish production from farming doubled. Fish farming caused by damage to ocean habitats resulting
produces a quarter of all fish and shellfish eaten by from fish farming.
humans, and, as global population increases, fish (B) Fish farming has some potential both for
farming will probably become even more important in increasing global fish supplies and for
supplying human protein needs. Some experts threatening those very supplies.
(10) even argue that in addition to helping to compensate (C) Fish farming is destined to supply ever-larger
for the decline in ocean harvests, fish farming will percentages of human protein needs.
restore wild populations by relieving pressure on ocean (D) The high catch rates for several types of wild-
fisheries. There is, however, little if any evidence that caught fish overshadow the advances made by
fish farming will restore ocean fishery stocks. fish farming.
(15) The complexity of production systems leads to an (E) Because of their diet, carnivorous fish are
underlying paradox: fish farming is a possible solution, more expensive and difficult to farm than
but also a potential contributing factor, to the continued noncarnivorous fish.
decline of ocean fishery stocks worldwide.
In the first place, the more intensive forms of fish 8. There is information in the passage sufficient to answer
(20) farming, oriented toward high-volume production, which one of the following questions?
threaten the sustainability of ocean fisheries through (A) How does the escape of species not native to a
water pollution and ecological disruption. Intensive farm’s region result in habitat destruction?
fish farming usually involves the enclosure of fish in a (B) What sort of shellfish is most commonly raised
secure system; population densities are typically high, on fish farms?
(25) resulting in the generation of large amounts of waste (C) Approximately how many kilograms of fish, on
and increased potential for the spread of pathogens. average, does a wild salmon consume during its
Habitat destruction through the spread of untreated lifetime?
waste, the escape of species not native to the farm’s (D) What proportion of the fish and shellfish eaten by
region, or contamination by new pathogens can all humans is produced on fish farms?
(30) ensue, seriously damaging ocean and coastal resources (E) How long does it take for niche markets for
and, ultimately, wild fishery stocks. wild-caught fish to have an appreciable effect on
Even more important, intensive farming of many wild populations?
species of fish requires large inputs of fish meal and fish
oil in order to supply fatty acids that vegetable matter 9. Which one of the following is closest to the meaning of
(35) lacks or essential amino acids that it inadequately the phrase “relieving pressure on ocean fisheries” as used
supplies, like lysine and methionine. For the ten species in the middle of the first paragraph?
of fish most commonly farmed, an average of 1.9
kilograms of wild fish is required for every kilogram of (A) making up for the scarcity of wild-caught fish
fish produced. Of the ten species, only three—catfish, (B) supplementing the incomes of people who make
(40) milkfish, and carp—require less fish input than is a living from ocean fishing
eventually harvested, while the farming of carnivorous (C) causing fewer wild fish to be caught
species like salmon has a very high input-to-output (D) reducing the amount of income to be earned from
ratio. Although some defenders of fish farming contend ocean fishing
that predatory carnivores in the wild consume even (E) reducing overall demand for fish and fish
(45) more fish than they would on a farm, farming of such products
carnivorous species requires up to 5 kilograms of wild
fish for every kilogram of fish produced.
Expanding farm production does have the potential
to alleviate some of the pressure on wild fishery stocks.
(50) For example, increasing the farm production of fish GO ON TO THE NEXT PAGE.
like salmon can reduce prices, deterring investment in
fishing fleets and, over time, reducing fishing efforts.
Similarly, other farmed fish like tilapia and channel
catfish provide alternatives to ocean fish like cod and
(55) haddock. Nonetheless, even these benefits may in
the end be lost because niche markets have started to
develop for several species of wild-caught fish, causing
their catch rates to remain high even as the production
of viable farmed substitutes has increased.

PrepTest 87
-4-

10. The information in the passage most strongly supports 12. The statements in the passage provide the most support
which one of the following statements? for the view that the author believes which one of the
following?
(A) Any further decline in ocean fishery stocks would
not be caused entirely by human activities. (A) Most farmed fish are carnivorous.
(B) The best way to reduce the price of wild-caught (B) Farmed and wild fish consume the same foods.
fish is to put a farmed variety of the same (C) Pollution is currently more damaging to wild fish
species on the market. populations than is overfishing.
(C) If fish farming does not continue to increase, (D) Market forces can either encourage or discourage
then it is unlikely that worldwide human protein overfishing of ocean fisheries.
needs can be met. (E) The market for wild-caught fish is likely to
(D) Most consumers do not perceive a difference in remain a small one.
taste between wild-caught and farmed varieties
of the same species of fish. 13. The information in the passage most strongly supports
(E) The use of wild fish to meet the nutritional needs which one of the following statements?
of farmed fish could result in the overfishing of
worldwide fish stocks. (A) Wild fish require different nutrients than farmed
fish do.
11. The views put forward in the passage conform most (B) It is more profitable to farm species such as
closely to which one of the following principles catfish, milkfish, and carp than to farm species
governing new methods of food production? such as salmon.
(C) The farming of tilapia and channel catfish
(A) They should be employed only if they do not produces as much environmental damage as the
result in major changes in consumer demand for farming of cod and haddock.
foods produced using traditional methods. (D) A growing number of consumers are choosing
(B) They should be employed if they are wild-caught fish of certain species in preference
economically more efficient than existing to farmed fish.
methods and their use will not result in business (E) Noncarnivorous fish in the wild typically do
failures or loss of jobs. not consume more food than their farmed
(C) They should not be employed if they will counterparts do.
ultimately result in a net decrease in food of the
type produced. 14. The author of the passage would be most likely to agree
(D) They should be employed if they promote with which one of the following statements regarding
diversification in the types of foods available pollution caused by fish farming?
for human consumption and help to bring about
environmental improvement. (A) Such pollution is problematic primarily because
(E) They should be employed only if they replace of the genetic mutations it would cause in wild
other methods that have undesirable ecological fish that live near fish farms.
effects. (B) Such pollution contributes to coastal degradation
more than any other type of pollution.
(C) Such pollution is less likely to occur if fish
farming enterprises do not pursue intensive
production methods.
(D) Such pollution cannot be prevented by fish
farmers alone without the involvement of
government agencies.
(E) Such pollution is, however harmful, unlikely to
prevent fish farming from eventually relieving
the pressure on wild fishery stocks.

GO ON TO THE NEXT PAGE.

PrepTest 87
-5-

Criminal courts frequently rely on accomplice witnesses: jurors may presuppose that accomplice
witnesses (witnesses who testify regarding the role witnesses and jailhouse informants offer their testimony
of an alleged co-conspirator in a crime) and jailhouse (60) as atonement rather than deducing that external factors
informants (witnesses who provide testimony based made it expedient to give the testimony.
(5) on information obtained while incarcerated) for
prosecutorial information. Typically the testimony 15. Which one of the following most accurately expresses
provided by such cooperating witnesses includes the main point of the passage?
information garnered through the witnesses’
conversations with the accused, which can include a (A) Evidence obtained through coercion, whether
(10) purported confession to the crime. that coercion results from positive pressure or
Information from a cooperating witness is often negative pressure, can never be regarded as
provided in exchange for a reduced sentence or some reliable.
other incentive. This kind of inducement creates a (B) Numerous considerations suggest that the
situation that is highly conducive to evidence courts’ reliance on the testimony of accomplice
(15) fabrication on the part of the cooperating witness. In witnesses and jailhouse informants may result in
fact, one recent study concluded that lying informants convictions based on false testimony.
are rarely prosecuted and therefore have much to gain (C) Studies show that jurors give undue weight
and little to lose by testifying falsely. to confession testimony, a fact that may be
While courts have recognized the unreliable nature explained by people’s general tendency to
(20) of evidence obtained through bartered testimony, they ignore situational factors in explaining the
have held that safeguards are in place to adequately behavior of others.
protect the accused against a conviction based on false (D) Traditional legal arguments offered in support
testimony. These safeguards allow effective cross- of permitting the testimony of accomplice
examination of a cooperating witness and enable the witnesses and jailhouse informants are based on
(25) jury to consider a witness’s motivations. However, a set of assumptions that numerous studies cast
these safeguards do not always provide protection. doubt on.
There are cases in which prosecutors merely imply (E) There is substantial evidence to indicate that the
to cooperating witnesses that they will receive an testimony of accomplice witnesses and jailhouse
incentive in exchange for testimony. In such cases, the informants is heavily influenced by incentives
(30) exchange between prosecution and witness does not offered by the prosecution.
have to be disclosed to the jury.
In addition, psychological research on confession 16. Which one of the following questions is explicitly
testimony—confessions obtained by investigators addressed in the passage?
directly from the accused—reveal further problems (A) Do jurors give the same weight to confession
(35) with bartered testimony. This research indicates that evidence provided by a cooperating witness as
jurors give undue weight to confession evidence when they do to confession evidence provided directly
rendering guilt decisions. This effect is especially by the defendant?
notable in cases where jurors are aware that a defendant (B) To what extent are prosecutors and investigators
has been offered an incentive in exchange for a limited in their ability to offer incentives to
(40) confession. This is particularly relevant here because accomplice witnesses and jailhouse informants
if people have difficulty realizing the effect that an in exchange for testimony?
incentive can have on a defendant’s behavior, they may (C) Is the bartered testimony of an accomplice
also fail to realize the effect that an incentive may have witness any more or less reliable than the
on a cooperating witness’s behavior. bartered testimony of a jailhouse informant?
(45) A common psychological phenomenon may account (D) How common is the prosecution of cooperating
for jurors’ superficial examination of confession witnesses who knowingly provide false
evidence. Studies show that people tend to explain the testimony?
behavior of others in terms of internal dispositions or (E) To what extent do jurors vary in their ability to
attitudes as opposed to external, situational factors. In discern when a witness is lying?
(50) one study, regardless of whether confession evidence
was obtained via negative pressure (threats of worse
treatment and/or harsher punishment) or positive
pressure (promises of better treatment and/or leniency),
mock jurors viewed a confession as evidence that the GO ON TO THE NEXT PAGE.
(55) defendant committed the crime because “only a guilty
person would confess to such a crime.” The same logic
can be applied to the testimony of cooperating.

PrepTest 87
-6-

17. The author would be most likely to agree with which one 20. The author mentions the research on confession
of the following statements about the courts’ reliance on testimony (fourth paragraph) primarily in order to
the testimony of cooperating witnesses?
(A) reveal a potential problem for the author’s
(A) It encourages unacceptably harsh treatment of analysis
prisoners by investigators and prison officials. (B) make an unfavorable comparison to a study cited
(B) It fails to recognize that cooperating witnesses earlier in the passage
may fear retaliation from defendants they testify (C) justify a conclusion regarding jurors’ treatment of
against. evidence provided by cooperating witnesses
(C) It frequently places an unfair burden on jurors. (D) question the relevance of jury decision-making
(D) It is justified only in cases in which the processes to the issue of the courts’ reliance on
prosecution has little other evidence against a the testimony of cooperating witnesses
defendant. (E) contrast the way in which jurors evaluate
(E) It likely leads to some convicted criminals’ evidence provided by a defendant with the
receiving sentence reductions that are way jurors evaluate evidence provided by a
unwarranted. cooperating witness

18. According to the third paragraph, current safeguards may 21. Which one of the following most accurately describes
be inadequate to protect a defendant from a cooperating how the final paragraph functions in the passage?
witness’s fabricated testimony because
(A) It attempts to explain a phenomenon discussed in
(A) current safeguards are designed to protect the the previous paragraph.
rights of witnesses rather than the rights of (B) It attempts to call into question a claim made in
defendants the previous paragraph.
(B) current safeguards fail to recognize the unreliable (C) It summarizes the problems discussed in the
nature of testimony that is obtained via coercion previous two paragraphs.
(C) juries may not be made aware that a cooperating (D) It outlines potential solutions to the problems
witness expects to receive an incentive from the discussed in the previous four paragraphs.
prosecution in exchange for testimony (E) It expands upon a proposal made in the first
(D) jurors tend to view the testimony of cooperating paragraph.
witnesses as more reliable than the testimony of
defendants
(E) prosecutors are typically not penalized for
offering incentives to cooperating witnesses in
exchange for testimony GO ON TO THE NEXT PAGE.

19. In using the phrase “jurors’ superficial examination


of confession evidence” {first sentence of the final
paragraph), the author most likely means to refer to
jurors’
(A) failure to properly take into account the factors
that may lead an individual to give confession
evidence
(B) failure to distinguish between confession
evidence offered by a defendant and confession
evidence offered by a cooperating witness
(C) lack of expertise necessary to accurately
evaluate confession evidence
(D) unwillingness to assess the veracity of a witness
who offers confession evidence
(E) tendency to disregard confession evidence when
it conflicts with other evidence presented at trial

PrepTest 87
-7-

Passage A is adapted from a book by a music historian, their music to sway public opinion in favor of their
and passage B from an anthropology journal. patrons. Their songs invoked specific public values and
described their patrons’ adherence to them, making the
Passage A griot a blend of community historian, storyteller,
Many commentators have described the blues (60) spokesperson, and ultimately, guardian of norms
musician of the United States as an extension of the and culture. Despite the griots’ public loudness,
griot of West Africa, yet one could hardly find two these performances and the prestige they brought
performers with less in common from a sociological their patrons required griots to be sensitive to Wolof
(5) perspective. Griots were the historians of their community values and conceptions of correct
communities, representatives of time-honored (65) social conduct.
traditions, the preservers of lore and cultural identity.
They took these traditions and transformed them into 22. Which one of the following is a principal purpose of
song, and as a result often enjoyed great status in their each passage?
(10) communities. In societies that lacked libraries and
museums, official documents and archives, the griot’s (A) to explain how a musical tradition can replace
song filled many of the roles that these institutions official institutions
serve in other societies. (B) to reveal the paradoxical nature of the
The blues musician, in contrast, honed a music relationship between a culture’s values and the
(15) of personal expression, often reflecting a lack of artists who help perpetuate those values
connection to the broader streams of society, evoking (C) to compare two closely related musical traditions
feelings of alienation and anomie. Slavery caused this (D) to explore the relationship between the social
terrible disjunction. Slavery destroyed in large part the standing of a group of musicians and the music
(20) historical continuities that made the griot’s art possible. they produce
Blues music was, in many ways, a response to this (E) to criticize a characterization of a particular
deprivation. culture
And here we encounter the fundamental tragedy
of the blues and one of the sources of its unparalleled 23. Based on the information in passage A about blues and
(25) symbolic power. For the music sings of small, the information in passage B about the music of Wolof
everyday details of individual lives. But behind this griots, which one of the following can most reasonably
façade always sits a larger reality, invariably unspoken, be inferred?
but no less present for this silence. Separated from the (A) Both types of music were drawn from feelings of
social institutions that gave life its meaning and alienation within a social structure.
(30) resonance within their traditional societies, African (B) Each type of music was created in response to a
Americans struggled to find substitutes for what cultural loss.
was lost within the smaller cosmos of their personal (C) Both types of music served to preserve the
relationships and daily life. Blues music reflected this traditions of their societies.
dynamic, gave it powerful poetic expression. (D) Both types of music were derived from earlier
(35) From this perspective, the perennial themes of blues African musical traditions.
music—heartache and hardships—capture in a personal (E) Each type of music is characterized by
dimension the larger social truth. subject matter that is typically drawn from a
circumscribed set of themes.
Passage B
Fifteenth-century Portuguese explorers observed a
stratified social hierarchy in the Wolof culture of
(40) Senegal, with a high-status noble sector (géer) and
low-status caste groups (ñeeño). Wolof elites of the day GO ON TO THE NEXT PAGE.
ranked ñeeño in six subcastes, the lowest of which was
griot.
Griots alone specialized in the spoken word.
(45) Raising one’s voice in public was considered
inappropriate for socially prominent people, but griots,
considered unmarriageable outside their caste, shouted
and sang their patrons’ praises to crowds of people,
often with a drum, and always with great eloquence.
(50) At community gatherings, griots accompanied
their patrons, with whom they had usually inherited
a close relationship through generations of service.
Reciting vivid histories about the brave deeds of their
patrons’ family ancestors and singing praises about
(55) their exemplary work and daily conduct, griots used

PrepTest 87
-8-

24. The attitude displayed in passage A toward blues 27. A difference in the way in which the two passages use the
musicians and the attitude displayed in passage B toward term “griot” is that
griots can both be characterized as
(A) passage A uses the term to refer to both musicians
(A) admiration for the musicians’ ability to represent and other performers whereas passage B uses
personal struggle symbolically the term to refer only to musicians
(B) unease about the musicians’ role in preserving a (B) passage A uses the term only to refer to a type of
social structure musician whereas passage B also uses the term
(C) envy of the musicians’ artistic contributions to refer to a social class
(D) approval of the musicians’ role as community (C) passage A uses the term to refer to both
historians contemporary and historical musicians whereas
(E) respect for the musicians’ artistry passage B uses the term to refer only to
historical musicians
25. Passage B indicates that which one of the following (D) passage B uses the term to refer to musicians who
claims made in passage A about griots was not true of perform only at community gatherings whereas
griots in fifteenth-century Wolof culture? passage A uses the term to refer to musicians
who also perform for small groups
(A) They were representatives of time-honored (E) passage B uses the term to refer only to
traditions. musicians employed by nobles whereas passage
(B) They preserved the cultural identity of their A uses the term to refer to musicians who have a
societies. variety of employment arrangements
(C) They often enjoyed great status in their
communities.
(D) They served a function analogous to that served
by libraries in other societies.
(E) Their art was made possible by the communal
values of their societies.

26. Passage B suggests that which one of the following was


true of fifteenth-century Wolof society?
(A) The society’s sense of shared communal values
was beginning to unravel.
(B) Public shouting and loud singing were acceptable
only among lower social classes.
(C) People who served as guardians of societal norms
and culture generally enjoyed high social status.
(D) Powerful nobles relied on numerous methods to
preserve their social status.
(E) Only members of the highest social class had a
conception of correct social conduct.

S T O P
IF YOU FINISH BEFORE TIME IS CALLED, YOU MAY CHECK YOUR WORK ON THIS SECTION ONLY.
DO NOT WORK ON ANY OTHER SECTION IN THE TEST.

PrepTest 87
-9-

SECTION II
Time—35 minutes
26 Questions

Directions: Each question in this section is based on the reasoning presented in a brief passage. In answering the questions, you
should not make assumptions that are by commonsense standards implausible, superfluous, or incompatible with the passage. For
some questions, more than one of the choices could conceivably answer the question. However, you are to choose the best answer;
that is, choose the response that most accurately and completely answers the question and mark that response on your answer sheet.

1. In situations where it is difficult to make informed 2. Biologist: Some small animals will instinctively go limp,
decisions about products, consumers should be “playing dead” when caught by a predator. But it
provided with the relevant information. The difficulty of is hard to see how playing dead can have survival
determining whether a food product contained nutritious value in this situation. The predator means to eat
ingredients was resolved by requiring food manufacturers the animal just the same, whether or not it plays
to print nutritional information on their products’ dead.
packaging. Similarly, many consumers are interested in
Which one of the following, if true, would most help to
conserving energy, and since there is no easy way for
resolve the apparent paradox described by the biologist?
consumers to determine how much energy was required
to manufacture a product, _______. (A) Many small animal species will play dead
when surprised by a loud noise or unexpected
Which one of the following most logically completes the
movement.
argument?
(B) Predators often leave their food in a hiding place
(A) consumers who are informed of the amount of rather than eating it immediately.
energy used to produce a product should choose (C) A small animal is more likely to play dead when
energy efficient products caught by a predator if the predator species is
(B) manufacturers should use less energy while common in the area.
producing products (D) Most predators prey upon a variety of species,
(C) providing consumers with information about not all of which play dead when caught.
the energy used to produce a product would (E) Many small animal species that do not play dead
reduce the impact of fossil fuels on our lives and are capable of fighting off predators.
economy
(D) consumers should demand products that require 3. Food columnist: Only 2 percent of imported seafood is
less energy to produce subjected to health safety inspections. So if you
(E) manufacturers should be required to label their want to increase the likelihood that the seafood
products with information about the amount of you buy will be safe to eat, you should buy only
energy used to produce those products domestic seafood.
The answer to which one of the following questions
would most help in evaluating the food columnist’s
argument?
(A) Do the health safety inspections detect all health
risks present in the seafood that is inspected?
(B) What kinds of health risks can seafood pose?
(C) What percentage of imported food other
than seafood is subjected to health safety
inspections?
(D) What percentage of domestic food other
than seafood is subjected to health safety
inspections?
(E) What percentage of domestic seafood is subjected
to health safety inspections?

GO ON TO THE NEXT PAGE.

PrepTest 87
- 10 -

4. Dog owner: In general, large dogs need less intensive 6. Principle: If someone makes an error, it is unethical for
exercise than smaller dogs to stay fit. A dog that a coworker to use that error to his or her own
is not exercised at the level of intensity it needs advantage.
is more apt to be troublesome than one that is. So Application: Because Mark used his coworker Rashmi’s
for any apartment dweller who has limited time clients’ e-mail addresses to advance his own
to give a dog exercise but who wants to have a career, his action was unethical.
dog, a large dog is less likely to be troublesome
than a small one. Which one of the following, if true, most helps to justify
the above application of the principle?
Which one of the following is an assumption the dog
owner’s argument requires? (A) Mark had the e-mail addresses of Rashmi’s
clients only because he had copied them from
(A) An apartment dweller who has limited time to Rashmi’s directory while she was at lunch.
exercise a dog is unlikely to want to own a dog. (B) A coworker of Rashmi and Mark had access to
(B) Providing a dog with more intensive exercise Rashmi’s clients’ e-mail addresses and shared
requires more time than providing a dog with them with Mark.
less intensive exercise. (C) Rashmi offered to help Mark develop a client
(C) At least some apartment dwellers who have base by sharing her own clients’ e-mail
limited time to exercise a dog should not own a addresses with him.
dog. (D) Mark had access to Rashmi’s clients’ e-mail
(D) Of dogs owned by apartment dwellers, those addresses only because she unintentionally left
that live in large apartments are less likely to them visible in an e-mail that she sent to both
be troublesome than those that live in small Mark and her clients.
apartments. (E) Mark happened upon a list of many of the e-mail
(E) In general, the more often a dog gets exercise, the addresses of Rashmi’s clients while conducting
more likely it is that the dog will stay fit. market research.

5. One theory concerning the importance of vitamin C in 7. Kevin: My barber shop sells an herbal supplement that,
the human diet holds that vitamin C plays a crucial role according to my barber, helps prevent baldness
in the production and maintenance of the body’s supply because it contains an enzyme that blocks the
of collagen, an important protein occurring almost formation of a chemical compound that causes
exclusively in connective tissue and bones. For this people to lose hair.
reason, some doctors believe that vitamin C can be useful Sabine: That’s simply not true. The fact is, your barber
in treating the symptoms of some common illnesses. makes money by convincing people to buy that
Which one of the following is most strongly supported product.
by the information above? Sabine’s argument is most vulnerable to criticism on the
(A) Some doctors believe that there are illnesses that grounds that it
affect the state of connective tissue or bones. (A) discounts scientifically plausible evidence merely
(B) Some doctors believe that vitamin C is the only because the person offering it is not a scientist
substance that produces and maintains collagen. (B) takes for granted that a product will be harmful
(C) Some doctors believe that strengthening if it is sold on the basis of an unsubstantiated
connective tissue and bones increases the body’s claim
ability to use certain vitamins. (C) rejects an explanation without proposing an
(D) Some doctors believe that use of vitamin C is the alternative explanation
most effective treatment for certain common (D) draws a conclusion about someone’s motives for
illnesses. making a particular claim without providing
(E) Some doctors believe that any illness that can be evidence that any such claim was actually made
ameliorated with vitamin C causes deterioration (E) rejects a claim merely because the person making
of connective tissue and bones. the claim stands to benefit by doing so

GO ON TO THE NEXT PAGE.

PrepTest 87
- 11 -

8. Analyst: When Johnson attacked his opponent by quoting 10. Researchers have found that most people’s bodies
her out of context, his campaign defended this attack make an enzyme, CYP2A6, that plays a crucial role in
by claiming that the quote was even more politically eliminating nicotine, the addictive drug in cigarettes,
damaging to her in context. But those who run his from the body. Smokers whose bodies make the most
campaign clearly do not believe this. They have common form of this enzyme tend to smoke more than
since had plenty of chances to refer to the quote in its those whose bodies make some other form of it. Why?
proper context but continue to quote it out of context. Well, the faster nicotine is eliminated from one’s body,
the sooner one will crave another cigarette, and _______.
Which one of the following principles, if valid, most
strongly supports the analyst’s reasoning above? Which one of the following, if true, most effectively
completes the explanation above?
(A) In criticizing an opponent, political campaigns will
pursue the line of attack they believe to be most (A) the most common form of CYP2A6 is the one
politically damaging. that most rapidly eliminates nicotine from the
(B) In criticizing an opponent, political campaigns do body
not use techniques that they would find (B) most people whose bodies make the rarest form
objectionable if used against their candidate. of CYP2A6 do not smoke at all
(C) In criticizing an opponent, political campaigns are (C) if one’s body does not make CYP2A6, nicotine
expected by voters to make sure that the quotes to will still be eliminated, although very slowly
which these campaigns refer are not taken out of (D) the greater the quantity of CYP2A6 that
context. one’s body makes, the faster nicotine will be
(D) In criticizing an opponent, political campaigns will eliminated
not be strongly criticized as long as the words (E) helping to eliminate nicotine is not the only
attributed to their opponent were actually said by function that CYP2A6 serves
their opponent.
(E) In criticizing an opponent, political campaigns will 11. Unlike other mechanical devices, the clock did not evolve
avoid using techniques that leave their candidate from the simple to the complex. The earliest clocks were
open to effective counterattacks. also the most complicated. This is because early clocks
were used primarily to predict astronomical phenomena,
9. Ellen: A group of economists and ecologists recently though the mechanisms they used for this purpose
estimated the economic value of Earth’s biosphere’s incidentally enabled one to keep track of time. Gradually
“essential services,” such as climate regulation, food, the timekeeping functions became more important and
and water supplies, at $33 trillion annually. the astronomical ones diminished.
We should therefore make protection of the
Which one of the following is most strongly supported
biosphere a high priority.
by the information above?
Santiago: I’m uncomfortable with the idea of calculating the
biosphere’s dollar value in order to justify protecting (A) Present-day clocks are of no use in the prediction
it. Such an approach implies that the biosphere’s of astronomical phenomena.
most important value lies in the “services” it (B) The mechanisms used to predict astronomical
provides us. phenomena in at least some clocks were more
complicated than most more recent mechanisms
On the basis of their dialogue, it can most reasonably be used for this function.
concluded that Ellen and Santiago disagree over the truth (C) Clocks used only for keeping time do not differ
of which one of the following statements? appreciably in their mechanical complexity.
(A) Estimating the dollar value of the biosphere’s (D) The mechanisms that the earliest clocks used
essential services is an appropriate way of providing to predict astronomical phenomena were more
a rationale for making protection of the biosphere complicated than the mechanisms used for
a high priority. timekeeping functions in some more recent
(B) The biosphere’s most important value lies in clocks.
something other than the services it provides to (E) Interest in predicting astronomical phenomena
human beings. has declined steadily since the invention of the
(C) Calculating the dollar value of the biosphere’s first mechanical clocks.
essential services is the most effective way to
ensure that protecting the biosphere is treated as a
matter of urgency.
(D) The idea that the dollar value of the biosphere’s
essential services can be accurately calculated is GO ON TO THE NEXT PAGE.
unrealistic.
(E) Calculating the dollar value of the biosphere’s
essential services implies that the biosphere’s most
important value lies in the services it provides
to human beings.

PrepTest 87
- 12 -

12. Regina: The additional revenue obtained from leasing 14. Biologists are mistaken in thinking that the fossil
government-owned toll bridges to private record provides direct evidence of the course of human
investors will be allocated to the transportation evolution. Fossils cannot be interpreted objectively:
budget, so the leases will not be used to reduce the physical characteristics by which they are classified
shortfalls in other budget areas. invariably reflect the models the paleontologists wish to
Amal: But allocating new revenue to transportation test. For example, classifying a pelvis as human because
will free up existing transportation funds for it supported an upright posture requires taking for granted
use in other areas. Thus, the new revenue will that bipedalism distinguished early hominids from apes.
nonetheless help reduce budget shortfalls in other Which one of the following most accurately expresses
areas. the overall conclusion of the argument?
Regina and Amal disagree over whether (A) No early apes had pelvises that would support an
(A) there will be shortfalls in budget areas other than upright posture.
transportation (B) The claims made by evolutionary theorists cannot
(B) the amount of money currently allocated to be objectively tested.
transportation is adequate (C) The fossil remains of some early hominids are
(C) new revenue from leasing government-owned toll difficult to distinguish from those of apes.
bridges should be allocated to transportation (D) The fossil record does not directly reveal the
(D) new revenue allocated to transportation will course of human evolution.
result in existing transportation funds being (E) Paleontologists’ classifications of fossils are
reallocated to other areas always influenced by the theories that these
(E) leasing government-owned toll bridges to private scientists are testing.
investors will be financially beneficial to the
government 15. The better we understand the behavior and ecological
niche of an endangered species, the better chance we
13. The use of ordinary dictionaries in interpreting the law is have of saving it. And the more individuals of a species
justified in the same way that chemists use the periodic we study, the better we understand it.
table. The periodic table is a convenient source of Therefore, _______.
agreed-upon background information that can be usefully Which one of the following most reasonably completes
applied to the problem on which a chemist is working. the argument?
In the same way, ordinary dictionaries can be useful to a
legal interpreter in resolving terminological issues. (A) many endangered species will become extinct
before we have the knowledge that is necessary
Which one of the following, if true, most seriously to save them
weakens the argument? (B) continued reduction of wildlife habitat will make
(A) The periodic table lists the properties of the the preservation of many endangered species
elements, and presents them in a pattern impossible
to represent relations between them, while (C) knowledge that contributes to saving endangered
an ordinary dictionary mostly just gives an species becomes harder to get as species become
alphabetical ordering to the words it defines. more endangered
(B) There is wide agreement about the data on the (D) to save endangered species it is more important
periodic table, while disagreements between the to acquire the right kind of knowledge than to
definitions in different ordinary dictionaries are take action
likely to be relevant to legal interpretation. (E) the impact of human study of endangered species
(C) The use of a periodic table as a reference source is sometimes more harmful than beneficial
actually came much later in history than the use
of ordinary dictionaries to describe the meanings
of words.
(D) The periodic table contains only a relatively
small amount of information that could, in GO ON TO THE NEXT PAGE.
theory, be memorized, while the information in
an ordinary dictionary is likely to be too large
for any one person to know all at once.
(E) The periodic table is used primarily by
chemists, while ordinary dictionaries are not
used primarily by legal scholars and legal
interpreters.

PrepTest 87
- 13 -

16. Art may make the world more beautiful, but one should 18. Some killer whales eat fish exclusively, but others also
choose a career in some profession other than art. eat seals. Different groups of killer whales “chatter” in
Whether and how much artists get paid is determined by distinct dialects, and the dialects of seal-eating killer
subjective evaluations by viewers or audiences of their whales are recognizably different from those of killer
work. It is unacceptable for one’s pay to be determined whales that do not eat seals. Harbor seals use their ability
by subjective evaluations of one’s work. to distinguish between different killer-whale dialects
to avoid seal-eating killer whales. Marine biologists
The reasoning in the argument is most vulnerable to
hypothesize that young harbor seals start with an aversion
criticism on the grounds that the argument
to all killer whales but then learn to ignore those that do
(A) takes for granted that people should choose not eat seals.
careers solely on the basis of how much they
Which one of the following, if true, provides the
pay
strongest support for the biologists’ hypothesis?
(B) takes for granted that a work of art will be
considered beautiful either by everyone or by no (A) Killer whales that eat seals also eat other marine
one mammals that are similar in size to seals.
(C) overlooks the possibility that one’s pay in (B) Unlike harbor seals, which can hear killer-whale
any profession involves a certain degree of chatter even at great distances, most fish cannot
subjective evaluation hear that chatter, even close at hand.
(D) overlooks the possibility that some artists are (C) When mature harbor seals first listen to the
paid very well recorded chatter of killer whales that eat only
(E) treats a criterion that must be satisfied in order for fish but whose dialect is unfamiliar, the seals
a career choice to be a good one as a criterion rapidly swim away from the sound.
that will ensure that a career choice is a good (D) Young harbor seals show no natural aversion to
one any seal predators other than killer whales.
(E) If a fish-eating killer whale mistakenly attacks
17. Critic: Vampires have traditionally been symbols of a harbor seal, that seal, if it survives, will
pure evil. Recently there has been a trend in subsequently avoid all killer whales that chatter
entertainment of humanizing vampires. This is in the attacker’s dialect, but other harbor seals
unfortunate. The overall trend in entertainment will not.
toward moral complexity is a good thing. But evil
exists in the world, and the vampire myth is one
of the most powerful representations of that.
The claim that the overall trend in entertainment toward
GO ON TO THE NEXT PAGE.
moral complexity is a good thing plays which one of the
following roles in the critic’s argument?
(A) It states a principle used to support the conclusion
of the argument.
(B) It places limits on how broadly the conclusion of
the argument should be generalized.
(C) It justifies the need for the argument’s being
given.
(D) It provides a hypothesis that is rejected in the
conclusion of the argument.
(E) It is the conclusion of the argument.

PrepTest 87
- 14 -

19. Trainer: An athlete developed lower back pain after 20. Some literary theorists argue that since literary works
a strenuous athletic competition. For several are expressions of ideology, it is naive to view them as
days, she tried to overcome the pain by daily embodying a distinct aesthetic value to a greater or lesser
stretching, but the pain continued. Then, on the degree. But these theorists evaluate particular literary
advice of a friend, she used a heating pad. Within works as being ideological expressions that are more or
a few days the pain was gone. This shows that the less interesting and successful. Therefore, these theorists
use of heating pads is generally more effective at succumb to the view they wish to undermine.
relieving lower back pain than stretching is.
The claim that the literary theorists evaluate particular
The trainer’s argument is vulnerable to criticism on each literary works as being ideological expressions that are
of the following grounds EXCEPT: more or less interesting and successful plays which one
of the following roles in the argument?
(A) It fails to consider that even if the use of heating
pads is more effective at relieving lower back (A) It is presented as evidence for the conclusion that
pain than stretching is, it may be much less it is naive to view literary works as embodying
effective at helping to heal the underlying injury a distinct aesthetic value to a greater or lesser
responsible for the pain. degree.
(B) It fails to consider the fact that lower back pain (B) It is presented as evidence against the claim that
resulting from athletic competitions often it is naive to view literary works as embodying
disappears after several days regardless of any a distinct aesthetic value to a greater or lesser
attempts to relieve it. degree.
(C) It fails to consider that the athlete’s experience (C) It is a conclusion for which the claim that it is
regarding the effectiveness of different methods naive to maintain that literary works embody
of relieving lower back pain may not have been a distinct aesthetic value to a greater or lesser
representative of that of the general population. degree is offered as evidence.
(D) It overlooks the possibility that the effectiveness (D) It is presented as evidence for the conclusion that
of different methods of relieving lower back the literary theorists succumb to the view they
pain may vary substantially depending on the wish to undermine.
underlying cause of the lower back pain. (E) It is presented as evidence against the claim that
(E) It overlooks the possibility that there might be literary works are expressions of ideology.
ways of stretching that are much more effective
at relieving lower back pain than were the ways
the athlete tried.

GO ON TO THE NEXT PAGE.

PrepTest 87
- 15 -

21. City official: Landowners must clear the snow from the 23. Critic: Journalists should have reasonable knowledge
sidewalks along the edge of their property by 24 of statistics. If not, they can make errors that
hours after the end of a snowstorm. The city has misinform the public. For example, based on a
the right to clear any sidewalk that is still covered clearly flawed interpretation of polling data, one
more than 24 hours after a snowstorm’s end, and journalist erroneously reported that a certain
whenever it does so, it will bill the landowner candidate would win an election.
for the service. All landowners whose sidewalks
The pattern of reasoning in the critic’s argument is most
have not been cleared within 48 hours of the
similar to that in which one of the following arguments?
end of a snowstorm will receive citations, which
always result in fines unless the landowners can (A) Before being issued a driver’s license, an
demonstrate extenuating circumstances. applicant should demonstrate an understanding
of the environmental effects of driving a fossil
If all of the official’s statements are true, which one of
fuel-powered car and be aware of the steps
the following must be true?
that drivers can take to lessen that impact.
(A) If the city clears a sidewalk of snow 50 hours Otherwise, there will be an unlimited increase in
after the end of a snowstorm, the owner will be the amount of carbon dioxide released into the
billed for the service and will receive a citation. atmosphere.
(B) All landowners who fail to clear their sidewalks (B) All elementary schools should have recess time.
by 24 hours after the end of a snowstorm will be Otherwise, children can incur health risks due
billed by the city for snow removal. to insufficient exercise. As a case in point, in
(C) All sidewalks in the city will be cleared of snow a school that had eliminated recess in favor of
within 50 hours of the end of a snowstorm. increased academic hours, the students were
(D) Nearly all landowners who do not clear their less physically fit than before the elimination of
sidewalks within 48 hours after the end of a recess.
snowstorm will be fined. (C) International travelers should have to be
(E) Landowners who can demonstrate extenuating processed through border security at each
circumstances will not be billed by the city for frontier they cross. The inconvenience this
snow removal service. might cause is something travelers must tolerate.
Otherwise, safety would be sacrificed for mere
22. Critic: Almost all of the paintings from our city’s art convenience.
movement share two characteristics: bold (D) All public officials should study ethics prior to or
brushwork and a sharp contrast of light and while holding their official position. The study
shadow. The only ones that do not share these of ethics would enable officials to anticipate the
characteristics are abstract paintings, which— moral pitfalls that characterize public service as
because they are nonrepresentational—do not well as to withstand the tendency to use public
depict light or shadow. However, the most service for private gain.
famous painting from our city—Blue Irises— (E) Toy manufacturers should be required to
cannot be considered part of the city’s art record the place and date a toy was made on
movement. For while this painting displays bold the toy itself. Otherwise, if the information is
brushwork, it does not exhibit a sharp contrast of recorded on the package only, the purchaser
light and shadow. can easily discard the package, thus losing this
information, which would make it impossible to
Which one of the following is an assumption required by
respond to manufacturer recalls.
the critic’s argument?
(A) In spite of its title, Blue Irises is
nonrepresentational.
(B) Blue Irises does not depict any shadows at all.
(C) Blue Irises is not an abstract painting. GO ON TO THE NEXT PAGE.
(D) All of the nonrepresentational paintings ever
produced in the critic’s city display bold
brushwork.
(E) All of the paintings from the critic’s city with
a sharp contrast of light and shadow emerged
from the city’s art movement.

PrepTest 87
- 16 -

24. Editorialist: Landis, one of this city’s top elected officials, 26. People should patronize businesses that meet high ethical
recently spent $10,000 to redecorate his office. standards, and the news media should help them to
Many people believe that if Landis used city patronize those businesses. Therefore, when a business
funds, then he misused public money, thereby performs a notably ethical action, the news media should
violating his official duties. But Landis is guilty publicize that fact, for hearing of a business’s ethical
of such violation regardless of the money’s conduct is often enough to motivate people to patronize
source. Spending $10,000 so frivolously is that business.
clearly immoral when so many people in our city
Which one of the following, if true, most seriously
live in poverty.
weakens the argument?
The editorialist’s conclusion follows logically if which
(A) Some businesses that have high ethical standards
one of the following is assumed?
do not actually meet those standards.
(A) The money Landis used was not his own money. (B) Meeting high ethical standards is primarily a
(B) It is immoral to spend money on luxury items matter of refraining from unethical behavior.
when there are people who lack basic necessities. (C) It is relatively easy for a business to meet its
(C) Landis knew about or participated in the decision ethical standards if it does not set them very
to redecorate his office. high.
(D) Every public official has an official duty never to (D) The news media is more likely to publicize
perform immoral actions. a business’s unethical conduct than it is to
(E) Had Landis not spent the money redecorating the publicize a business’s ethical conduct.
office, it would have been used to help alleviate (E) Some businesses that meet high ethical standards
poverty in the city. would not do so if they could not remain
profitable while meeting those standards.
25. Katelin says that we will be hit by a major snowstorm
tomorrow. So she probably believes that tomorrow’s
antique car show will be canceled, for it will certainly not
be held if we are hit by a major snowstorm.
The questionable pattern or reasoning in the argument
above is most clearly parallel to that in which one of the
following?
(A) Jorge says that given today’s pollution levels,
many species of migratory birds will become
extinct. Thus, he probably believes that the
extinctions will occur, since it is widely known
that pollution might not be reduced at all.
(B) Bo says that the soil in his backyard is poorly
drained, and since raspberry bushes will not
grow well in soil that is poorly drained, Bo
probably believes that raspberry bushes will not
grow well in his backyard.
(C) Wanda says that no form of coercive force is ever
justified. Thus, since most people who say this
believe that government should be abolished,
Wanda probably believes that government
should be abolished.
(D) My chemistry professor says that most chemists
are good at math. So, my chemistry professor
might believe that most chemists are good at
chess, since most people who are good at math
are also good at chess.
(E) Dr. Bowder says that eating garlic increases one’s
alertness. So, since she knows that whatever
improves one’s circulation increases one’s
alertness, Dr. Bowder probably believes that
eating garlic increases one’s circulation.
S T O P
IF YOU FINISH BEFORE TIME IS CALLED, YOU MAY CHECK YOUR WORK ON THIS SECTION ONLY.
DO NOT WORK ON ANY OTHER SECTION IN THE TEST.

PrepTest 87
- 17 -

SECTION III
Time—35 minutes
26 Questions

Directions: Each question in this section is based on the reasoning presented in a brief passage. In answering the questions, you
should not make assumptions that are by commonsense standards implausible, superfluous, or incompatible with the passage. For
some questions, more than one of the choices could conceivably answer the question. However, you are to choose the best answer;
that is, choose the response that most accurately and completely answers the question and mark that response on your answer sheet.

1. Minh: This film director’s newest works are very 3. Columnist: Although it is our civic duty to protect the
predictable. He’s pillaging his own catalog, but population against hazards to public health, we
with diminishing returns. Each film is simply a should not reroute high-tension power lines away
repetition of his earlier ones. from heavily populated areas. This is because our
Natalie: You mistake the films’ startling sameness for limited resources should be devoted to protecting
evidence of a lack of creativity. It would be more the population only against well-substantiated
accurate to say that he ultimately creates strong threats to public health.
new works from the same core elements, and The conclusion of the columnist’s argument can be
these works are thus original. properly drawn if which one of the following is
The dialogue most strongly supports the claim that Minh assumed?
and Natalie agree on the truth of which one of the (A) Public health would be damaged by the loss of
following statements about the director’s newest works? electric power.
(A) They share many features with his earlier films. (B) Proponents of expensive safety measures with
(B) They constitute evidence that he is pillaging his respect to high-tension power lines ignore
own catalog. economic realities.
(C) They are nothing more than repetitions of the (C) Scientific evidence exists for causal links
director’s earlier films. between various modern practices and threats to
(D) They are less original than his earlier films. public health.
(E) They provide evidence of the director’s creativity. (D) No investigation of the effects of high-tension
power lines has established any health threat to
2. Campaign manager: In campaign speeches, our candidate people.
has been reluctant to reveal all the unpleasant (E) Rerouting high-tension power lines away from
consequences of the policies he endorses, but heavily populated areas would hinder our ability
this lapse can be justified. There is no way to get to study the effects of power lines on people.
elected while being fully candid. And it is vitally
important that our candidate get a chance to help
implement a political agenda that is very positive.
Which one of the following principles, if valid, most GO ON TO THE NEXT PAGE.
helps to justify the reasoning in the campaign manager’s
argument?
(A) Ethically questionable acts can sometimes be
justified by their good consequences.
(B) It is better to fail while behaving ethically than to
succeed by resorting to unethical measures.
(C) Legitimate ends obtained through illegitimate
means become illegitimate.
(D) It is possible to accomplish one’s agenda even if
one is not trusted.
(E) Voters can best be trusted to make good decisions
when they are fully informed.

PrepTest 87
- 18 -

4. The coat patterns of large cat species correspond to the 6. The best way to increase the blood supply in the city of
habitats in which those species live and hunt. Species Pulaski is to encourage more donations by people who
with spotted coats are at home in trees and dappled are regular blood donors. A study conducted in two other
forests, while species living in the open plains, such as cities—Moorestown and Fredricksburg—indicates that
lions, have plain coats. The only anomaly is the cheetah, it is difficult and expensive to attract first-time blood
a spotted cat that lives in the open savannah. donors, and that many sporadic donors are reluctant
to give more often. But officials in those cities had
Which one of the following, if true, most helps to
considerable success convincing many of their cities’
explain the anomaly described above?
regular donors to increase their donation frequency.
(A) Unlike all other large cat species, cheetahs’
Which one of the following, if true, would do most to
hunting strategy does not rely on stealth but
undermine the conclusion of the argument above?
instead relies purely on speed.
(B) Of all the large cat species, cheetahs most often (A) Increasing blood donation by regular donors
have their prey stolen from them by larger in Moorestown and Fredricksburg produced a
predators. significant increase in the blood supply in those
(C) Because they have wide paws with cities.
semiretractable claws, cheetahs are not able to (B) The pool of potential blood donors in
climb upright trees. Moorestown and Fredricksburg contained
(D) Unlike lions, cheetahs are typically solitary proportionally fewer frequent blood donors than
hunters. does the pool of potential donors in the city of
(E) Unlike all other large cat species, cheetahs are Pulaski.
unable to roar. (C) A follow-up study in Moorestown and
Fredricksburg showed that long-term frequency
5. As the current information explosion forces the print of blood donation among regular donors
media, television, and the Internet to compete for public remained higher after the promotional campaign
attention and for advertiser and subscriber dollars, than it had been before the campaign.
journalistic standards are lowered. Consequently, we (D) In the city of Pulaski, the number of sporadic
are increasingly bombarded with inaccurate and trivial blood donors is significantly greater than the
information. number of regular blood donors.
(E) Almost all of the regular blood donors in the city
Which one of the following most accurately expresses
of Pulaski are already giving blood as frequently
the conclusion of the argument?
as is medically safe.
(A) The drawbacks of the information explosion now
outweigh its benefits. 7. Advertisement: Honey Oat Puffs cereal is made with
(B) People are more and more subjected to whole-grain oats. Health experts agree that
insignificant and unreliable information. whole-grain oats are among the most healthful
(C) Journalistic standards have fallen in recent years. foods, far more healthful than most of the foods
(D) One result of the current information explosion in a typical diet. So most people would have
is fierce competition among the print media, more healthful diets if they ate Honey Oat Puffs
television, and the Internet for both money and for breakfast.
public attention.
The reasoning in the advertisement’s argument is most
(E) If journalists returned to earlier journalistic
vulnerable to criticism on the grounds that the argument
standards, the significance and reliability of
news stories would increase. (A) infers that a food will have a certain property
simply because one of the food’s ingredients has
that property
(B) treats a property that is sufficient to make a food
healthful as a property that a food must have in
order to be healthful
(C) confuses two distinct meanings of the word
“healthful”
(D) concludes that a food contributes to health simply
because that food tends to be part of the diets of
healthy people
(E) contains a premise that presupposes the truth of
the conclusion

GO ON TO THE NEXT PAGE.

PrepTest 87
- 19 -

8. Astronomer: Conditions in our solar system have 10. Giant ground sloths began disappearing from the
probably favored the emergence of life more than Americas about 10,000 years ago, around the time that
conditions in most other solar systems of similar the last ice age ended, and are now extinct worldwide.
age. Any conceivable form of life depends on Scientists had thought that these sloths failed to adapt
the presence of adequate amounts of chemical to climate changes, but they are now coming to believe
elements heavier than hydrogen and helium, and that it was the arrival of human beings shortly before
our sun has an unusually high abundance of these that ice age ended that was responsible for the sloths’
heavier elements for its age. disappearance.
Which one of the following most accurately describes Which one of the following, if true, provides the most
the role played in the astronomer’s argument by the support for the scientists’ new belief?
claim that any conceivable form of life depends on
(A) Scientists have not found any physical evidence
chemical elements heavier than hydrogen and helium?
to support the idea that giant ground sloths were
(A) It is a statement for which no evidence is hunted to extinction.
provided and that is part of the evidence offered (B) Species of smaller tree-dwelling sloths continue
for the argument’s only conclusion. to live throughout South and Central America.
(B) It is a statement for which no evidence is (C) Their large size made the giant ground sloths less
provided and that is offered as support for adaptable than most other ground sloths.
another statement that in turn is offered as (D) Giant ground sloths are not the only large
support for the conclusion of the argument as a mammals that began to disappear from the
whole. Americas around 10,000 years ago.
(C) It is a statement for which some evidence is (E) One type of giant ground sloth survived on
provided and that itself is offered as support for isolated islands until human beings arrived there
the conclusion of the argument as a whole. well after the last ice age.
(D) It is the conclusion of the argument as a whole
and is supported by another statement for which
support is offered.
(E) It is one of two conclusions in the argument,
neither of which is offered as support for the GO ON TO THE NEXT PAGE.
other.

9. Political organizer: Our group needs to assemble at


least 30 volunteers if Marcia Garson is to have
a chance of winning the election, since she will
win only if the public is fully informed about her
record. To fully inform the public, at least 30 of
our people must campaign for her, but we simply
cannot afford to pay people for this work.
Which one of the following most accurately expresses
the conclusion drawn in the political organizer’s
argument?
(A) Marcia Garson will probably not be elected.
(B) The political organizer’s group cannot afford to
pay people to campaign for Marcia Garson.
(C) If winning the election is to be a possibility for
Marcia Garson, the political organizer’s group
needs to bring together at least 30 volunteers.
(D) If the public is not fully informed about Marcia
Garson’s record, she will not win the election.
(E) At least 30 people from the political organizer’s
group need to campaign for Marcia Garson in
order to fully inform the public about her record.

PrepTest 87
- 20 -

11. Studies have shown that those who take daily doses of 12. The Discourses, a work attributed to the ancient Stoic
vitamin C are less likely to contract colds than are those philosopher Epictetus, is believed to have been compiled
who do not. Thus, if a person contracts a cold, he or she from Epictetus’s lectures by his student Arrian. Some
probably does not take daily doses of vitamin C. claim that Arrian himself authored The Discourses
and falsely attributed authorship to Epictetus. That is,
The questionable reasoning in the argument above most
however, highly unlikely. Epictetus’s views were well
closely parallels that in which one of the following?
known by his contemporaries in Roman society; if
(A) To be classified as a hurricane, a tropical storm Arrian tried to pass off his own views as Epictetus’s, this
in the Atlantic Ocean must have sustained wind deception would soon be exposed.
speeds of over 74 miles (119 kilometers) per
Which one of the following, if true, most strengthens the
hour. So tropical storms with sustained wind
argument?
speeds at about that level are not hurricanes.
(B) Everyone who ate the tuna salad prepared in the (A) The Discourses would have received very little
cafeteria has contracted botulism. So those who attention in Roman society if that work had not
ate other dishes prepared in the cafeteria will been attributed to Epictetus.
probably also get this disease. (B) Some of the philosophy that modern scholars
(C) Laboratory rats given an extremely low-calorie believe Arrian subscribed to can be found in The
diet live up to 20 percent longer than do those Discourses.
given a more ordinary diet. So most people (C) Arrian in some ways modeled himself on the
could extend their life spans by eating a great historian Xenophon, who had written works that
deal less than they usually do. he falsely attributed to the philosopher Socrates.
(D) People who train diligently to play poker can (D) Arrian had high status in Roman society, which
achieve a certain level of proficiency, so those he knew would be undermined if he were shown
players owe their success to hard work alone. to be dishonest.
(E) Automobile engines that undergo regular oil (E) During his long career, Arrian was a soldier, a
changes are more likely to be problem free consul of Rome, and a governor in Turkey.
than are those that do not. So automobiles with
engine problems probably have not had regular 13. A study tested the performance of 70 pilots, half of
oil changes. whom chose to go on a reduced-calorie diet. Those
who did not diet performed well, while pilots who were
dieting performed worse, although they had performed
well before they started their diets. The average level
of impairment for the dieters was approximately equal
to that caused by consuming two alcoholic drinks on an
empty stomach.
Each of the following, if true, contributes to an
explanation of the study results described above
EXCEPT:
(A) Dieters often become preoccupied with worries
about their weight losses and calorie intakes.
(B) Many of the pilots, including both dieters and
nondieters, consumed alcohol before the tests,
and dieting increases vulnerability to alcohol’s
effects.
(C) Reduced-calorie dieting makes most people more
vulnerable to irritability and fatigue.
(D) Many of the pilots chose to go on a diet
because they were curious about whether one’s
weight affects one’s piloting skills.
(E) Whereas alcohol has no effect on the level of
glucose (a nutrient vital to brain function) in the
bloodstream, dieting lowers the glucose level.

GO ON TO THE NEXT PAGE.

PrepTest 87
- 21 -

14. Guam has 40 times more spiders than nearby islands 16. If a novelist is popular he or she can vividly imagine
have. Biologists argue that this is a consequence of the large numbers of characters, each with a personality and
accidental 1940s introduction into Guam of the brown attitudes that are completely different from those of the
tree snake, which by the 1980s had eliminated ten of others and from those of the novelist. Such a writer also
twelve native bird species. The biologists attribute the will be capable of empathizing with people who have
spider population’s increase to the loss of bird species, goals completely different from his or her own and so
because many birds prey on spiders and some use will have some doubts about the genuine value of his or
spiderwebs in constructing nests. her own desires.
Which one of the following, if true, most strengthens the If the statements above are true then each of the
biologists’ conclusion? following could also be true EXCEPT:
(A) Birds compete with spiders for insect prey. (A) Some novelists who can vividly imagine large
(B) The biologists counted spiderwebs as a means of numbers of characters with attitudes and
estimating the spider population. personalities completely different from those of
(C) Spiderwebs are more prevalent on Guam than on the others are not popular.
nearby islands. (B) Some novelists are incapable of empathizing with
(D) The two bird species remaining on Guam have people whose goals are completely different
proliferated since the arrival of the brown tree from their own.
snakes. (C) Some people who lack the ability to empathize
(E) Brown tree snakes have proven difficult to with those who have goals completely different
eradicate on Guam. from their own are popular novelists.
(D) No people who have doubts about the value of
15. If the glee club pays cash today to rent the equipment it their own desires are incapable of empathizing
needs for next Saturday’s party, its usual rental agency with people who have goals that are completely
will accept a much lower price than the club would pay different from their own.
for renting the equipment next Saturday. The club’s party (E) Most writers who have doubts about the value of
committee knows the costs involved but nonetheless will what they desire are popular novelists.
wait until next Saturday to pay. The committee clearly
cares little about saving money.
Which one of the following assumptions is required by
the argument above?
GO ON TO THE NEXT PAGE.
(A) At least some influential members of the club’s
party committee are interested in the savings
available by renting the equipment today rather
than renting it next Saturday.
(B) Most members of the club’s party committee are
aware of the glee club’s financial situation.
(C) At least one rental agency will not accept a lower
price for equipment rental today than it would
accept next Saturday.
(D) The club’s party committee has a number of
responsibilities that take priority over saving
money.
(E) There is enough cash available to the glee club
today to pay for the equipment rental at today’s
lower price.

PrepTest 87
- 22 -

17. After monitoring blood levels of lycopene (a nutrient 19. Most movie critics believe that sentimentality detracts
found in some fruits and vegetables) in 1,000 middle- from aesthetic value. But these critics are wrong, since
aged study participants over a 12-year period, researchers the reason they hold this belief is that sentimentality
found that participants with low levels of lycopene pervades so many movies that its absence makes a movie
were more than twice as likely as those with high levels more interesting to frequent movie-goers like themselves.
to have a stroke during that period. Clearly, lycopene It is like someone whose food is usually prepared with
reduces the risk of stroke. a certain type of flavoring concluding that the flavoring
itself detracts from the quality of the food.
Which one of the following, if true, most weakens the
argument? The reasoning in the argument is flawed in that the
argument
(A) Most fruits and vegetables that are rich in
lycopene also contain high levels of several (A) is based solely on an inappropriate appeal to
other nutrients that are thought likely to reduce authority
the risk of stroke. (B) rejects a position merely on the grounds that
(B) Countries in which people consume substantial someone who argues for it has an ulterior
quantities of lycopene-rich fruits and vegetables motive
generally have lower rates of stroke than other (C) takes a necessary condition for a movie’s
countries. being of high aesthetic value to be a sufficient
(C) Middle-aged people typically have lower condition for this
lycopene levels than young adults. (D) concludes that a view is false merely on the
(D) Study participants with high levels of lycopene grounds of how people came to believe it
consumed, on average, twice the quantity of (E) takes what is sufficient for diminishing the
fruits and vegetables as those with low levels of quality of a work to be necessary for doing so
lycopene.
(E) There was wide variation in lycopene levels
among study participants.

18. In order to cut costs and thereby maximize his profit, Mr. GO ON TO THE NEXT PAGE.
Kapp used inferior materials in constructing the library.
Whether legal or not, it was clearly wrong for him to do
so. For, as an experienced and knowledgeable builder,
he must have realized that his action would put people at
serious risk.
The argument’s conclusion is properly drawn if which
one of the following is assumed?
(A) Any knowledgeable and experienced builder
would realize that using the inferior construction
materials that Mr. Kapp used would put people
at serious risk.
(B) An action can be wrong from a moral standpoint
without necessarily being legal.
(C) Mr. Kapp made a large profit from the
construction and sale of the building.
(D) It is wrong to knowingly put people at serious
risk for the sake of profit.
(E) Mr. Kapp either knew or ought to have known
that using the inferior materials he used to build
the library would put people at serious risk.

PrepTest 87
- 23 -

20. Business owner: Although allowing coal mining in our 22. Laurie: In a democracy, public art should bring people
region would create new jobs, we can expect the together either by expressing a consensus on a
number of jobs in the region to decrease overall subject or by helping people to reconcile their
if it is permitted. Many local businesses depend differences and to recognize that no single
on our region’s natural beauty, and the heavy opinion is definitive. Since contemporary public
industrial activity of coal mining would force art creates only acrimony, it has failed in its task.
most of them to close. Elsa: If people hold radically different opinions, public
Which one of the following most accurately describes art should emphasize that. No art form can do the
the role played in the business owner’s argument by the impossible, which is what you are asking for.
claim that many local businesses depend on the region’s Laurie’s and Elsa’s statements provide the most support
natural beauty? for holding that they disagree about
(A) It is given as direct evidence for a statement (A) what types of public art are most characteristic of
that is used to support the argument’s overall contemporary democracies
conclusion. (B) whether it is possible in a democracy to create
(B) It is a premise that is offered as direct support for public art that people with radically different
the overall conclusion of the argument. opinions can enjoy and support
(C) It is an intermediate conclusion offered as direct (C) what the criterion of success for public art in a
support for the argument’s main conclusion. democracy should be
(D) It is the overall conclusion drawn in the (D) whether contemporary public art creates only
argument. acrimony
(E) It is a hypothesis for which evidence is explicitly (E) whether it is wise for contemporary public art to
offered, but it is not itself intended to support help achieve a consensus on a project
the argument’s overall conclusion.
23. Environmental ethicist: Since whooping cranes, unlike
21. Columnist: Obviously, money helps one satisfy one’s sandhill cranes, are endangered as a species, the
desires. However, people become less happy as survival of any one whooping crane is much
they become more wealthy. For, though wealth more important to the preservation of its species
allows one to satisfy desires one would not than the survival of any one sandhill crane is to
otherwise be able to, it invariably creates an the preservation of its species. Hence, we have
even greater number of desires that will not be a greater duty to protect the life of an individual
satisfied. whooping crane than we do to protect the life of
Which one of the following, if true, most strengthens the an individual sandhill crane.
columnist’s argument? The environmental ethicist’s reasoning conforms most
(A) Extreme wealth impedes the attainment of the closely to which one of the following principles?
highest level of happiness. (A) Any duty to protect the life of an individual
(B) The fewer unfulfilled desires one has, the happier organism is entirely independent of the duty to
one is. protect the species to which that organism belongs.
(C) One’s happiness tends not to increase each time a (B) The more important the survival of individual
desire is satisfied. members is to the preservation of a species,
(D) There are very few wealthy people who would the greater the duty to protect the lives of that
not prefer to be wealthier. species’ individual members.
(E) Satisfying one’s desires is not the only relevant (C) The fewer species an endangered species is
factor to one’s happiness. closely related to, the greater the duty to protect
that species.
(D) There is a greater duty to protect a species as a
whole than there is to protect any individual
member of that species.
(E) There is a greater duty to protect one individual
organism over another only if the former
organism is a member of an endangered species
and the latter organism is not.

GO ON TO THE NEXT PAGE.

PrepTest 87
- 24 -

24. All of the one-way streets in the city have dedicated 26. Employee: Vernon’s behavior in last month’s incident
bike lanes. City buses do not travel on any street with a was certainly unprofessional enough that our
dedicated bike lane. Parking is allowed only on streets company was justified in firing him. But several
that do not have a dedicated bike lane. City bus number higher-ranking employees whose behavior in
nine travels the full length of Batchelder Avenue. the incident was just as unprofessional haven’t
been fired and are treated as employees in good
If the statements above are true, which one of the
standing. So for the sake of consistency, the
following must also be true?
company must give Vernon his job back.
(A) Batchelder Avenue is a one-way street.
Which one of the following most accurately describes a
(B) Batchelder Avenue is not a one-way street.
flaw in the employee’s argument?
(C) Parking is allowed on Batchelder Avenue.
(D) Parking is not allowed on Batchelder Avenue. (A) illicitly using a key term in different senses
(E) Parking is not allowed on any street on which during the course of the argument
buses do not travel. (B) confusing behavior that is sufficient to justify an
action with behavior that is required to justify
25. Ecological terms like “invasive species” invoke human that action
cultural standards like belonging, citizenship, fair play, (C) offering as its primary evidence a premise that is
and morality and apply them to the natural world. These equivalent to the argument’s conclusion
terms can influence ecologists’ opinions of certain (D) treating behavior that can sometimes result in
organisms before any data is gathered about their a certain consequence as behavior that always
ecological impact. To prevent this, they should avoid results in that consequence
using such terms. (E) inferring that one specific response to a problem
is necessary without considering another equally
Which one of the following conforms most closely to the
supported response
principle illustrated above?
(A) Police should not use terms like “thief” when
talking to suspects no matter how strong
the evidence of their guilt, since suspects
are officially innocent until they have been
convicted by a court.
(B) Environmental regulators should not argue
publicly that particular substances are harmful,
since the government’s decisions about which
substances are harmful should be made by
legislators.
(C) Opinion writers should avoid using overly
unflattering language to describe their
opponents’ views on controversial issues, since
such language often leads readers simply to
discount the views of the writer that uses it.
(D) Software developers should refrain from using
psychological terms to describe the performance
of their programs, since this could influence
their expectations about how those programs
will handle unanticipated inputs.
(E) People should be careful when attributing
motives to another person in order to explain
that person’s behavior, since psychological
studies suggest that such attributions are
frequently incorrect.

S T O P
IF YOU FINISH BEFORE TIME IS CALLED, YOU MAY CHECK YOUR WORK ON THIS SECTION ONLY.
DO NOT WORK ON ANY OTHER SECTION IN THE TEST.

PrepTest 87
- 25 -

SECTION IV
Time—35 minutes
23 Questions

Directions: Each set of questions in this section is based on a scenario with a set of conditions. The questions are to be answered on
the basis of what can be logically inferred from the scenario and conditions. For each question, choose the response that most
accurately and completely answers the question and mark that response on your answer sheet.

Questions 1–5 1. Which one of the following could be the order in which
the six groups are evaluated?
The participants in an experiment designed to evaluate the
persuasive impact of expert witness testimony will be divided (A) judges, police officers, teachers, medical doctors,
into six groups, according to occupation—judges, lawyers, nurses, lawyers
medical doctors, nurses, police officers, and teachers. The (B) judges, teachers, medical doctors, lawyers, police
six groups will be evaluated separately, one after the other, in officers, nurses
accordance with the following conditions: (C) medical doctors, judges, police officers, nurses,
The teachers must be evaluated at some time before the teachers, lawyers
medical doctors and at some time before the nurses. (D) teachers, lawyers, police officers, nurses, medical
The medical doctors must be evaluated at some time doctors, judges
before the lawyers. (E) teachers, medical doctors, judges, police officers,
The nurses must be evaluated either immediately before nurses, lawyers
or immediately after the police officers.
The police officers must be evaluated at some time 2. Which one of the following groups could be evaluated
between the judges and the lawyers, regardless of sixth?
whether the judges are evaluated before the lawyers or (A) judges
after. (B) medical doctors
(C) nurses
(D) police officers
(E) teachers

GO ON TO THE NEXT PAGE.

PrepTest 87
- 26 -

3. If the lawyers are evaluated at some time before the 5. What is the minimum number of groups that must be
judges, which one of the following could be true? evaluated after the teachers?
(A) The judges are evaluated third. (A) five
(B) The medical doctors are evaluated fifth. (B) four
(C) The nurses are evaluated third. (C) three
(D) The police officers are evaluated fourth. (D) two
(E) The teachers are evaluated second. (E) one

4. If the police officers are evaluated fifth, which one of the


following must be true?
(A) The judges are evaluated third. GO ON TO THE NEXT PAGE.
(B) The lawyers are evaluated sixth.
(C) The medical doctors are evaluated second.
(D) The nurses are evaluated fourth.
(E) The teachers are evaluated first.

PrepTest 87
- 27 -

Questions 6–10 6. How many of the commercials are there any one of
which could be aired last?
A television station must determine the order in which five
commercials, each advertising a single product—fast food, (A) five
granola, pizza, sportswear, or trucks—are to be aired during (B) four
the five slots available in a commercial break. The order of the (C) three
commercials is subject to the following constraints: (D) two
The pizza commercial must be aired earlier than the (E) one
granola commercial.
The sportswear commercial must be aired earlier than the 7. If the truck commercial is aired earlier than the granola
truck commercial. commercial, then which one of the following CANNOT
The fast-food and sportswear commercials must be aired be true?
consecutively.
(A) The fast-food commercial is aired second.
(B) The granola commercial is aired fourth.
(C) The pizza commercial is aired first.
(D) The sportswear commercial is aired first.
(E) The truck commercial is aired fourth.

GO ON TO THE NEXT PAGE.

PrepTest 87
- 28 -

8. If the pizza commercial is aired immediately before the 10. If the granola and truck commercials are not aired
truck commercial, then which one of the following could consecutively, then which one of the following could be
be true? true?
(A) The truck commercial is aired third. (A) The fast-food commercial is aired last.
(B) The sportswear commercial is aired third. (B) The granola commercial is aired fourth.
(C) The pizza commercial is aired first. (C) The pizza commercial is aired third.
(D) The granola commercial is aired fourth. (D) The sportswear commercial is aired second.
(E) The fast-food commercial is aired second. (E) The truck commercial is aired fourth.

9. How many of the commercials are there any one of


which could be aired second?
(A) five GO ON TO THE NEXT PAGE.
(B) four
(C) three
(D) two
(E) one

PrepTest 87
- 29 -

Questions 11–17 11. Which one of the following could be the schedule of the
paintings shown in the four weeks, listed in order from
Over the course of four consecutive weeks, a gallery is going the first week to the fourth?
to show three oil paintings—Gold, Hanbok, and Ibex—and
three watercolor paintings—Ping, Roil, and Sails. Each (A) Gold and Roil; Hanbok and Sails; Ibex and Ping;
painting will be shown in at least one week, with exactly one Gold and Roil
oil painting and one watercolor painting shown per week, (B) Gold and Roil; Ibex and Sails; Hanbok and Ping;
subject to the following conditions: Gold and Roil
No painting can be shown in two consecutive weeks. (C) Ibex and Ping; Gold and Roil; Hanbok and Sails;
Gold cannot be shown in any week in which Ping is Gold and Roil
shown. (D) Ibex and Ping; Hanbok and Sails; Gold and Ping;
Hanbok must be shown in any week in which Sails is Hanbok and Roil
shown. (E) Ibex and Ping; Hanbok and Sails; Hanbok and Ping;
Hanbok cannot be shown earlier than the third week Gold and Roil
unless Ibex is shown in the first week.

GO ON TO THE NEXT PAGE.

PrepTest 87
- 30 -

12. If Sails is shown in the second week, then any of the 15. The schedule of the paintings shown in the four weeks is
following could be shown in the third week EXCEPT: completely determined if which one of the following is
true?
(A) Gold
(B) Hanbok (A) Gold is shown twice.
(C) Ibex (B) Hanbok is shown twice.
(D) Ping (C) Ping is shown twice.
(E) Roil (D) Roil is shown twice.
(E) Sails is shown twice.
13. Which one of the following must be false?
16. If Roil is shown in both the second and fourth weeks,
(A) Sails is shown in the first week. which one of the following could be true?
(B) Hanbok is shown in the second week.
(C) Roil is shown in the third week. (A) Gold is shown in the first week.
(D) Ibex is shown in the fourth week. (B) Hanbok is shown in the second week.
(E) Ping is shown in the fourth week. (C) Hanbok is shown in the fourth week.
(D) Ibex is shown in the second week.
14. If Gold is shown in the first week, which one of the (E) Ibex is shown in the fourth week.
following could be true?
17. Which one of the following, if substituted for the
(A) Ibex is shown in the third week. condition that Gold cannot be shown in any week in
(B) Ping is shown in the third week. which Ping is shown, would have the same effect in
(C) Roil is shown in the second week. determining the schedule of the paintings shown in the
(D) Roil is shown in the fourth week. four weeks?
(E) Sails is shown in the second week.
(A) Roil must be shown in any week in which Gold is
shown.
(B) Gold must be shown in any week in which Roil is
shown.
(C) If Ping is not shown in the first week, then Sails
must be shown no earlier than the third week.
(D) If Ping is shown in the first week, then Ibex must
also be shown in the first week.
(E) Hanbok cannot be shown in any week in which
Roil is shown.

GO ON TO THE NEXT PAGE.

PrepTest 87
- 31 -

Questions 18–23 18. Which one of the following could be the assignment of
volunteers for Friday and Saturday?
A charity is assigning volunteers to work at its booth at a fair
that runs three days: Thursday, Friday, and Saturday. Five (A) Friday: Lentz, Morse, Nuñez
volunteers are being assigned: Lentz, Morse, Nuñez, Pang, Saturday: Lentz, Morse, Nuñez
and Quinn. On each day, exactly three of the volunteers will (B) Friday: Lentz, Morse, Pang
work at the booth. The assignment of volunteers to days must Saturday: Lentz, Morse, Quinn
meet the following conditions: (C) Friday: Nuñez, Pang, Quinn
No volunteer works every day. Saturday: Lentz, Morse, Nuñez
On any day that Morse works, Lentz also works. (D) Friday: Nuñez, Pang, Quinn
Nuñez works on Friday. Saturday: Lentz, Morse, Pang
Pang does not work on Saturday. (E) Friday: Nuñez, Pang, Quinn
Saturday: Morse, Nuñez, Quinn

19. If Pang works on Friday, then any of the following could


be true EXCEPT:
(A) Lentz works on Friday.
(B) Morse works on Thursday.
(C) Morse works on Friday.
(D) Nuñez works on Thursday.
(E) Nuñez works on Saturday.

GO ON TO THE NEXT PAGE.

PrepTest 87
- 32 -

20. If Pang works on only one day, which one of the 22. If Morse works on Thursday, which one of the following
following must be true? must be true?
(A) Lentz works on Thursday. (A) Lentz works on Friday.
(B) Morse works on Saturday. (B) Morse works on Saturday.
(C) Nuñez works on Thursday. (C) Nuñez works on Saturday.
(D) Pang works on Friday. (D) Pang works on Thursday.
(E) Quinn works on Saturday. (E) Quinn works on Friday.

21. The assignment of volunteers to days is completely 23. If Morse works on only one day, which one of the
determined if which one of the following is true? following must be true?
(A) Lentz works on Thursday. (A) Lentz works on Friday.
(B) Morse works on Friday. (B) Morse works on Saturday.
(C) Nuñez works on Thursday. (C) Nuñez works on Saturday.
(D) Pang works on Friday. (D) Pang works on Thursday.
(E) Quinn works on Saturday. (E) Quinn works on Friday.

S T O P
IF YOU FINISH BEFORE TIME IS CALLED, YOU MAY CHECK YOUR WORK ON THIS SECTION ONLY.
DO NOT WORK ON ANY OTHER SECTION IN THE TEST.

PrepTest 87
- 33 -

SECTION I

1. D 8. D 15. B 22. D
2. B 9. C 16. D 23. E
3. A 10. E 17. E 24. E
4. E 11. C 18. C 25. C
5. A 12. D 19. A 26. B
6. C 13. D 20. C 27. B
7. B 14. C 21. A

SECTION II

1. E 8. A 15. C 22. C
2. B 9. A 16. C 23. B
3. E 10. A 17. B 24. D
4. B 11. D 18. C 25. B
5. A 12. D 19. A 26. B
6. D 13. B 20. D
7. E 14. D 21. A

SECTION III

1. A 8. A 15. E 22. C
2. A 9. C 16. C 23. B
3. D 10. E 17. A 24. B
4. A 11. E 18. D 25. D
5. B 12. D 19. D 26. E
6. E 13. D 20. A
7. A 14. A 21. B

SECTION IV

1. E 8. E 15. E 22. E
2. A 9. C 16. E 23. D
3. D 10. D 17. A
4. D 11. C 18. C
5. B 12. B 19. C
6. D 13. A 20. B
7. B 14. D 21. B

PrepTest 87
- 34 -

June 2019 Conversion Table

The table below should be used for converting raw scores to the 120–180 score scale for the June 2019 administration.
The table below shows the LSAT reported score that corresponds to each possible raw score.

Reported Raw Score


Score Lowest Highest
180 100 102
179 99 99
178 —* —*
177 98 98
176 97 97
175 —* —*
174 96 96
173 95 95
172 94 94
171 93 93
170 92 92
169 90 91
168 89 89
167 87 88
166 86 86
165 84 85
164 83 83
163 81 82
162 79 80
161 77 78
160 75 76
159 73 74
158 71 72
157 69 70
156 68 68
155 66 67
154 64 65
153 62 63
152 60 61
151 58 59
150 56 57
149 54 55
148 52 53
147 50 51
146 49 49
145 47 48
144 45 46
143 44 44
142 42 43
141 41 41
140 39 40
139 38 38
138 36 37
137 35 35
136 33 34
135 32 32
134 31 31
133 30 30
132 29 29
131 28 28
130 27 27
129 26 26
128 25 25
127 24 24
126 —* —*
125 23 23
124 22 22
123 —* —*
122 21 21
121 20 20
120 0 19

*There is no raw score that will produce this scaled score for this form.

PrepTest 87

You might also like